A truck parked on the street is 56 m from the base of the apartment building on the right. The angle of elevation from the truck to the top of the building is 40°. The parked truck is 49 m from the base of the apartment building on the left. The angle of elevation from the truck to the top of that building is 50°. Which building is taller?

Answers

Answer 1

Answer:

The taller building is the second building which is the apartment building on the left with an height of 58.4m

Step-by-step explanation:

This is a Trigonometric question. We solve this using the Tangent formula

tan θ = Opposite side/Adjacent side

We are given 2 buildings

Building 1

tan θ = Opposite side/Adjacent side

θ = Angle of Elevation = 40°

Distance from the base of the building = Adjacent = 56m

Height of the building = Opposite side = unknown

tan 40 = 0pposite/56m

Opposite = tan 40 × 56m

Opposite = 46.989579346m

Approximately = 47m

Height of Building 1 = 47m

Building 2

tan θ = Opposite side/Adjacent side

θ = Angle of Elevation = 50°

Distance from the base of the building = Adjacent = 49m

Height of the building = Opposite side = unknown

tan 50 = 0pposite/49m

Opposite = tan 50 × 49m

Opposite = 58.395926037m

Approximately = 58.4m

Height of Building 2 = 58.4m

The taller building is the second building which is the apartment building on the left with an height of 58.4m


Related Questions

Solving Quadratics



Solve for all values of x by factoring. x² + 4x + 5 = -6x + 5​

Answers

Answer: x=5 and x=-1

Step-by-step explanation:

X^2-4x-5=0

(X-5) (x+1)=0

X-5=0. And. X+1=0

8 gal to 9 1/3 gal write ratio in lowest form with whole numbers in numerator and denominator

Answers

Answer:

[tex]\bold{\dfrac{6}{7}}[/tex]

Step-by-step explanation:

Given ratio is:

8 gal to [tex]9\frac{1}3[/tex] gal

To find:

Ratio in Lowest form with whole numbers in numerator and denominator.

Solution:

First of all, let us convert [tex]9\frac{1}3[/tex] to simple [tex]\frac{p}{q}[/tex] form:

Formula:

[tex]a\dfrac{b}{c} = \dfrac{a\times c+b}c[/tex]

Using the formula, we get:

[tex]9\dfrac{1}{3} = \dfrac{9\times 3+1}{3} = \dfrac{28}{3}[/tex]

Now, let us have a look at the ratio:

[tex]8 : \dfrac{28}{3}[/tex]

Any ratio, [tex]p:q[/tex] can be written as [tex]\frac{p}{q}[/tex].

Writing given ratio as per above:

[tex]\dfrac{8}{\dfrac{28}{3}}\\\Rightarrow \dfrac{8 \times 3}{28}\\\Rightarrow \dfrac{2\times 3}{7}\\\Rightarrow \bold{\dfrac{6}{7}}[/tex]

Therefore, the answer is

[tex]\bold{\dfrac{6}{7}}[/tex]

the first half of food+the last quarter of door​

Answers

Answer:

For

Step-by-step explanation:

the first half of food + the last quarter of door

= F o o d + d o o r

= Fo + r

= For

Brandon wants to buy a new bike. The model he likes is on sale for 15% off its original price. His parents agree to pay for $50 of the cost. If the original cost of the bike is x dollars, which of the expressions below represent the amount Brandon has to pay?

Answers

Answer:

0.85x - $50

Step-by-step explanation:

Given that :

Brandon wants to buy a new bike. The model he likes is on sale for 15% off its original price. His parents agree to pay for $50 of the cost. If the original cost of the bike is x dollars, which of the expressions below represent the amount Brandon has to pay

Discount on price = 15%

Amount parents agreed to pay = $50

Cost of bike = x

Discount on bike = 0.15x

Cost after discount is removed = (x - 0.15x) = 0.85x

Parent pays $50 of cost price, amount Brandon will have to pay equals :

(0.85x - $50)

Answer:

0.85x - $50

Step-by-step explanation:

Find the sum: 3.85 + 6.29 =

Answers

Answer:

3.85 + 6.29 = 10.14

Hoped I helped

Step-by-step explanation:

3.85+6.29= 10.14

thank

A rectangular pool is twice as
long as it is wide. What are the
dimensions of the pool if the
perimeter is 48 yd?

Answers

Step-by-step explanation:

L=2B

P=2(L+B)

48=2(2B+B)

48=2(3B)

48=6B

B=8yd

L=2*8

=16yd

Answer:

length * width = 16 yd * 8 yd

or

length = 16 yd

width = 8 yd

Step-by-step explanation:

take the rectangular pool to have sides a (length) and 2a (width).

we are told that the length is as twice as its width, therefore,

Perimeter is given by (a + 2a) × 2 = 48 yd

Divide both sides by 2 ⇒ [tex]\frac{((a + 2a) * 2)}{2} = \frac{48 yd}{2}[/tex] ⇒ a + 2a = 24 yd

⇒3a = 24 yd

Divide both sides by 3 ⇒ [tex]\frac{3a}{3} = \frac{24 yd}{3}[/tex] ⇒ a = 8 yd

therefore, the width of the pool is 8 yd and its length is 16 yd (Twice as long as it is wide)

Given x=-3, y=6, and z=-4 x+y+(-1)=

Answers

Answer:

8

Step-by-step explanation:

x=3

y=6

to find x+y-1, sub in x and y

3+6-1 = 8

Answer: 8

Step-by-step explanation:

x+y+(-1), given x=3, y=6, and z=-4

---------

 x+y+(-1)

=(3)+(6)+(-1) ⇔ substitute x and y

=9+(-1) ⇔ add 3 and 6

=8

Hope this helps!! :)

Please let me know if you have any questions

There are nine teddy bears in the diagram below. How many TOTAL squares are
there?

Answers

Answer: 14

Step-by-step explanation: 9 small squares with one teddy bear in each, 4 medium squares with 4 bears in each medium square, and 1 big square for the entire thing.

The total number of square is 14

What is figure counting?

In the counting of figures, you have a shape or a figure. From the given shape you will have to identify a given known shape and count the number of times it is present in the given shape.

According to the question

9 small squares with one teddy bear in each,

4 medium squares with 4 bears in each medium square.

1 big square for the entire thing.

Total square = 9 + 4 + 1

                     = 14

Hence, the total number of square is 14

To learn more about counting figure from here

https://brainly.com/question/17456957

#SPJ2

20 POINTS!!! NEED ALL 4 ANSWERED!!!

Set the two sides equal to each other and solve for x!

① -2x + 81 = 61

② 9x - 58 = 5

③ 21 + 10x = 3x + 28

④ 9x - 42 = 2x

(I already put them in the correct forms based on each images But here's the images anyways, just in case.)

Answers

Answer:

1. x = 10  2. x = 7  3. x = 1  4. x = 6

Explanation:

1. -2x + 81 = 61 (Subtract 81 from 61)

  -2x = -20 (Divide)

   x = 10

2. 9x - 58 = 5 (Add 58 to 5)

   9x  = 63 (Divide)

   x = 7

3. 21 +10x = 3x + 28 (Rearrange expression)

   21 - 28 = 3x - 10x (Combine like terms)

   -7 = -7x (Divide)

    x = 1

4. 9x - 42 = 2x (Rearrange)

   9x - 2x = 42 (Combine like terms)

   7x = 42 (Divide)

   x = 6

What is the area of 6ft and 5ft

Answers

Answer:

30 ft

Step-by-step explanation:

6 x 5 = 30

you multiply 6 times 5 and you get 30

Step-by-step explanation:


Solve -4(x + 1) – 3 = -3(x - 4).

Answers

Answer:

x =-19

Step-by-step explanation:

[tex]-4(x + 1) - 3 = -3(x - 4).\\\\\mathrm{Expand\:}-4\left(x+1\right)-3:\quad -4x-7\\\\\mathrm{Expand\:}-3\left(x-4\right):\quad -3x+12\\-4x-7=-3x+12\\\\\mathrm{Add\:}7\mathrm{\:to\:both\:sides}\\-4x-7+7=-3x+12+7\\\\Simplify\\-4x=-3x+19\\\\\mathrm{Add\:}3x\mathrm{\:to\:both\:sides}\\-4x+3x=-3x+19+3x\\\\Simplify\\-x=19\\\\\mathrm{Divide\:both\:sides\:by\:}-1\\\frac{-x}{-1}=\frac{19}{-1}\\\\Simplify\\x=-19[/tex]

Answer:

We have this equation:

-4(x+1) - 3 = -3(x-4)

First of all, we solve the paretheses:

-4x - 4*1 - 3 = -3x - (-3*4)

-4x - 4 - 3 = -3x +  12

-4x -7 = -3x + 12

We sum 7 on both sides and 3x on both too:

-4x - 7 + 7 + 3x = -3x + 12 + 3x + 7

-4x + 3x = 12 + 7

-x = 19

x = -19

Now we can verify:

-4(-19+1) - 3 = -3(-19-4);

-4(-18) - 3 = -3(-23);

72 - 3 = 69 (TRUE)

Which of the following options correctly represents the complete factored form of the polynomial?

Answers

Answer:

B

Step-by-step explanation:

f(x)=(x-3)(x²+2x+2)

when solve for x using quadratic formula fo x in (x²+2x+2):

x=(-b±√b²-4ac)/2a  a=1,b=2,c=2

x=-1±i

f(x)=(x-3)(x+1-i)(x+1+i)

The complete factored form of the polynomial is F(x) = (x - 3)(x + 1 + i)(x + 1 - i).

To find the complete factored form of the polynomial F(x) = x³ - x² - 4x - 6, we can factor it using various methods such as synthetic division or factoring by grouping.

Factoring the polynomial, we get:

F(x) = (x + 3)(x - 1)(x + 2)

Comparing the factored form with the options provided:

A. F(x) = (x + 3)(x + 1)(x - 1)

This option does not match the factored form of the polynomial.

B. F(x) = (x - 3)(x + 1 + i)(x + 1 - i)

f(x)=(x-3)(x²+2x+2)

When solve for x using quadratic formula fo x in (x²+2x+2):

x=(-b±√b²-4ac)/2a  

x=-1±i

f(x)=(x-3)(x+1-i)(x+1+i)

So, the is the required factored form.

C. F(x) = (x - 3)(x + 1 + i)(x - 1 - i)

This option does not match the factored form of the polynomial.

D. F(x) = (x + 3)(x + 1 + i)(x + 1 - i)

This option does not match the factored form of the polynomial.

Learn more about Factor here:

https://brainly.com/question/15872577

#SPJ6

Hi May I know how to solve this?

Answers

Answer:

k = 3/4

Step-by-step explanation:

Parallel lines have the same slope so first, we need to find the slope of 8x + 6 = 2y. To do so, we can do the following:

8x + 6 = 2y

2y = 8x + 6

y = 4x + 3 (This is now in slope intercept form - y = mx + b where m = slope and b = y-int.)

Therefore, the slope is 4. Now, we can do:

3x = ky - 10

3x - 10 = ky

(3x / k) - (10 / k) = y

y = (3x / k) - (10 / k)  (where k ≠ 0)

Now that the second line is in slope-intercept form, we can set the two x terms equal to each other, therefore:

3x / k = 4x

3x = 4x * k

3x / 4x = k

k = 3/4

please answer and give reasoning. might give brainliest.

Answers

I think the answer is A but don’t count me on it

Answer:

A

Step-by-step explanation:

A contains √3 --> irrational number because 3 is not a square number

For other choices:

- All numbers in the form a/b (a,b are integer numbers) are rational numbers

- √36, √9, √4, √25, √49, √100 are integer numbers because (36, 9, 4,....) are square numbers

- All given decimal numbers can be written in the fraction form

0.5 = 1/2

39.77 = 39 + 77/100 = 3977/100

0.15151578 and 0.00010001 are finite decimals, 0.303030... is infinite repeating decimal, so technically they are rational numbers.

add r to q, then add s to the result

Answers

Answer:

(r+q)+s

Step-by-step explanation:

The parentheses make it clear that r and q need to be added first.

AB is a straight line

Work out the size of angle x

Answers

Wouldn’t you just subtract the 132 degrees from 180 degrees ?

On James’s MP3 player, he has 12 sad songs and 40 upbeat songs that he wants to put into playlists. He wants to have the same number of sad songs and upbeat songs in each playlist. What is the maximum number of playlists that he can create?

Answers

Answer:

4 playlists

:)

A large western state consists of 3129 million acres of land. Approximately 55% of this land is federally owned. Find the number of acres that are not federally owned

Answers

Answer:

1408.5 acres

Step-by-step explanation:

If 55% of the land is federally owned, then 100 - 55 = 45% of the land is not federally owned.

We need to find how many acres are not federally owned, in other words, we need to find how many acres represent 45% of the total 3129 acres. So, to do this we can apply a rule of three:

3129 acres ---- 100%

x acres     ------ 45%

[tex]x=\frac{(3129)(45)}{100} =3129(0.45)= 1408.05[/tex]

Therefore, the total of acres that are not federally owned are 1408.5 acres.

5. Which of the following is a multiple of 8?
A 4
C 20
B 8
D 100

Answers

Answer:

8

Step-by-step explanation:

8 can be divisible by 8. Therefore, it would be a multiple of 8.

The first multiple of a number is always that number.

All other options given are not multiples of 8.

Hope this helps.

B) 8 since 8 is one of the multiple of 8 and it’s the beginning of the multiples of 8

Evaluate the expression: 6:3+2x7

Answers

Answer:

15.41

Step-by-step explanation:

Can someone help me with this problem please??? If ƒ( x ) = x 2 + 1 and g( x ) = 3 x + 1, find 2 · ƒ(4). A. 18 B.34 C.65

Answers

Answer:  B  

Step-by-step explanation:

They've given you two functions  f(x) = x^2 + 1 and g(x) = 3x +1  And they are you to find 2 * f(4) then g(x) is not needed so input in  4 for the function f(x) and multiply the output by 2.

F(4) = 4^2 + 1

F(4) = 16 +1

f(4) =  17  

   17  * 2 = 34

Answer:

B.34

Step-by-step explanation:

[tex]f( x ) = x^ 2 + 1 \\ g( x ) = 3 x + 1\\\\f(4) \times 2 = ?\\f(4) = 4^2 +1\\\\=( 16+1) \times 2\\\\= 17\times 2\\\\= 34[/tex]

A recipe that makes 25 oatmeal cookies calls for 2.5 cups of oats and one cup of sugar. Jerry needs to make 195 cookies for his party. How many cups of oats will he need?

Answers

you would need 19.5 cups of oats and 7.8 cups of sugar because 195 divided by 25 is 7.8 so then you just multiply the both of them by that

Use the Distance Formula to answer the following questions. Round all of your answers to the nearest tenth (one decimal place). 2) Find the distance between the points (-3, 1) and (5,-2). Round your answer to the nearest teeth. ​

Answers

Answer:

distance= 9 units to the nearest

Step-by-step explanation:

using the formula

√(∆x)^2+(∆y)^2

√73

= 8.5

= 9 units

Answer:  8.5 units approximately

===========================================

Work Shown:

d = distance between (-3,1) and (5,-2)

[tex]d = \sqrt{(x_1 - x_2)^2 + (y_1 - y_2)^2}\\\\d = \sqrt{(-3-5)^2 + (1-(-2))^2}\\\\d = \sqrt{(-3-5)^2 + (1+2)^2}\\\\d = \sqrt{(-8)^2 + (3)^2}\\\\d = \sqrt{64 + 9}\\\\d = \sqrt{73}\\\\d \approx 8.5440037\\\\d \approx 8.5\\\\[/tex]

Alternatively, you can plot the two points on the same xy grid. Then form a right triangle with the two points as the endpoints of the hypotenuse. Use of the pythagorean theorem will result in getting the same approximate distance. The distance formula is effectively the same as the pythagorean theorem, but just in a different form.

i need help with this page​

Answers

Answer:

<abc=14

Step-by-step explanation:

<abc= 14 ; because if the whole thing is 90 and <CBD is already 76 then you subtract 90-76 to get the rest of the angle which is 14 degrees

Question attached below:

Thanks! :)

Answers

Answer:

(a) aₙ = 22 + 6(n - 1)

(b) The 14th row has 100 seats

Step-by-step explanation:

1. (a) Remember that an arithmetic series has the general formula a + (n - 1)d. Here a = the first term, and d = difference between first and second term, or in other words the difference between the nth and (n + 1)th terms.

aₙ = 22 + 6(n - 1)

This is our iterative rule for this arithmetic series, aₙ = 22 + 6(n - 1).

(b) Here n = the row number. We want to know the row number that has 100 seats. Let's equate the expression '22 + 6(n - 1)' to 100, and solve for n.

100 = 22 + 6(n - 1),

100 = 22 + 6n - 6,

100 = 16 + 6n,

6n = 84,

n = 84 / 6 = 14

=> the 14th row has 100 seats

The function h(x)=4/3x-1 represents the composite h(x)=f(g(x)).if f(x)=2x-1,what is g(x)

Answers

Answer:

[tex]g( x) = \frac{2}{3}x[/tex]

Step-by-step explanation:

Given:

Composite Function:

[tex]h( x)=f( g( x))[/tex]

Also,

[tex]h( x)=\frac{4}{3}x-1[/tex]

To find:

[tex]g( x ) = ?[/tex]

Solution:

First of all, let us learn about a composite function.

Composite function [tex]f( g( x))[/tex] means to write [tex]g( x)[/tex] in place of [tex]x[/tex] in the function [tex]f( x)[/tex].

Let [tex]g( x) = y[/tex]

So, [tex]f( g( x))[/tex] =  [tex]f( y )[/tex].

Therefore,

[tex]h( x) =f( g( x)) = f( y)[/tex]

So, [tex]f( y) = 2y-1[/tex]

We have to solve for [tex]y[/tex] in terms of [tex]x[/tex] to find the value of [tex]g(x)[/tex]:

[tex]\Rightarrow \frac{4}{3}x-1 = 2y-1\\\Rightarrow 2y=\frac{4}{3}x\\\Rightarrow y = \frac{2}{3}x[/tex]

Hence, the answer is [tex]g( x ) = \frac{2}{3}x[/tex].

Checking whether the answer is correct or not:

[tex]f( g( x)) = f( \frac{2}{3}x) = 2\times \frac{2}{3}x -1 = \frac{4}{3}x-1[/tex]

which is [tex]h( x)[/tex].

Hence, our answer [tex]g( x ) = \frac{2}{3}x[/tex] is correct.

The g(x) of the function is:

g(x) = (2/3)x

How to find what g(x) is?

A function is a relationship between inputs where each input is related to exactly one output.

Every function has a domain and codomain or range. A function is generally denoted by f(x) where x is the input.

We have:

h(x) = (4/3)x - 1

h(x) = f(g(x))

f(x) = 2x - 1.

Thus, we can say:

h(x) = f(g(x)) = 2*g(x) - 1

(4/3)x - 1 = 2g(x) - 1

(4/3)x - 1 + 1= 2g(x)

(4/3)x = 2g(x)

(4/3)x * 1/2 = g(x)

(2/3)x = g(x)

g(x) = (2/3)x

Therefore, g(x) = (2/3)x.

Learn more about function on:

brainly.com/question/29233399

#SPJ6

someone tell me the answers tho these please. Part A: Factor 3x2y2 − 2xy2 − 8y2. Show your work. (4 points) Part B: Factor x2 + 10x + 25. Show your work. (3 points) Part C: Factor x2 − 36. Show your work. (3 points)

Answers

Step-by-step explanation:

x2 + 10x + 25

x2 + 5x + 5x + 25

x(x + 5) + 5(x + 5)

(x + 5)(x+ 5) are the factors

x2 - 36

Both are squares so

x2 - (4)2


solve for x and y please

Answers

Answer:

Y=40

X=67

Step-by-step explanation:

In a cross line problem, one part of the cross, (such as 63°) is always the same on the other side. (like y+23° and 63°. This means Y=40 because 40+23 is 63.

To find x, you must know that 360° is all the way around in degrees. you know that 126° (63 +63) is already taken up. So you subtract 126 from 360 and then multiply the answer by 1/2 because you trying to find the bottom total.(360-126)=234(1/2)= 117 Lastly, you find X in the equation 2x-17=117.

x=67

Name the Reflection Rule that results in the following transformation

A(1,4), B(2, 7), C(9, 3) to

A'(1,-4), B'(2, -7), C'(9, -3)

Answers

Reflect across the x-axis because the rule is (x, -y)

3(x + 5) = 2(4x + 9) ​

Answers

Answer:

x=-3/5

Step-by-step explanation:

3(x + 5) = 2(4x + 9) ​

First, distribute:

3(x)+3(5)=2(4x)+2(9)

3x+15=8x+18

-5x= 3

x= -3/5

Other Questions
Life is good developed the "Good Karma" line of environmentally friendly 100% organic cotton apparel. The production of the Good Karma line is consisten with a_____orientation. a) societal marketing b) supplier. c) sales. d) production. e) philanthropic. what is eurozone ? in eu work Solve for . 5w-26=-4(w-7) Simplify your answer as much as possible. PLZ HELP A block of aluminum occupies a volume of 0.015 L and has a mass of 40.5 g. What is its density g The large negative free energy of hydrolysis of Phosphoenolpyruvate is largely due to _______________ of the enol pyruvate. Its AP stats and is the answer quantitative or qualitative or something I dont know I need help finishing a question. I used the explanation function on the program, but it didn't specify what to do to get the final answer. I have attached a picture."Find the real solutions. Use the quadratic formula and a calculator."At the last step, it said to use a calculator to see if the solutions satisfy the equation, but does that mean I need to plug in the whole thing for each x in the original equation? That would take a long time. I tried punching the "(1 plus or minus sqrt (1 + 160pi^2)) / 2" into my calculator, but the answers I got were wrong. Freeman Motors, a motorcycle manufacturer, had the following contingencies. Determine the appropriate accounting treatment for each of the situations Freeman is facing. a. Freeman estimates that it is reasonably possible but not likely that it will lose a current lawsuit. Freeman's attorneys estimate the potential loss will be $4,500,000. Describe the situation in a note to the financial statements. Do not disclose. Record an expense and a liability based on estimated amounts. b. Freeman received notice that it was being sued. Freeman considers this lawsuit to be frivolous. Describe the situation in a note to the financial statements. Do not disclose. Record an expense and a liability based on estimated amounts. c. Freeman is currently the defendant in a lawsuit. Freeman believes it is likely that it will lose the lawsuit and estimates the damages to be paid will be $75,000. Hello please help, thank you!! Vivian is getting a pet snake. She is choosing between the ball python and the corn snake. the ball python is 4 1/2 ft long and the corn snake is 45 in. long. Vivian wants the shorter snake. Which snake should she get? Show your work. (12 in. = 1 ft) Elsa's fish tank has 19 liters of water in it. She plans to add 4 liters per minute until the tank has more than 47 liters. What are the possible numbers ofminutes Elsa could add water?Use t for the number of minutes.Write your answer as an inequality George went to The Clothes Shack and bought 4 pairs of jeans for $30, 2 pairs of T-shirts for $15, and 3 pairs of shorts for $20. Write an expression that can be used to find the total amount that George spent. How much did George spend? what is described by the wavelength of a wave All questionshzbzbzbz John had seen people sprinkle salt on ice in the winter. The salt appeared to melt the ice. He decided to sprinkle salt on a larger block of ice. Then he added food coloring so he better could see the results. How did creativity help John in this experiment? A. It helped him use materials in a new way to help him gather data. B. It helped him reason through the results of his experiments. C. It helped him follow the steps of the procedure precisely. D. It helped him remember something from his past. cade ate 14 slices of pizza. He ate 4more than Dre. How many slices ofpizza did Dre eat? number (12+10-2) is? charles darwin is credited with outlining the principles of evolution by natural selection. what aspects of the scientific method can be identified in darwin's work Help please Im trying to help my child but Im not even sure. Determine whether the following contributions belong to Samuel Adams or George Washington.